Med Surg Exam 2

Lakukan tugas rumah & ujian kamu dengan baik sekarang menggunakan Quizwiz!

A client has had a kidney transplant performed for end-stage kidney disease. What type of immune response that T-cell lymphocytes perform is related to this type of surgery?

A cell-mediated response

Which instruction about insulin administration should a nurse give to a client?

"Always follow the same order when drawing the different insulins into the syringe."

he nurse is educating a patient about the benefits of fruit versus fruit juice in the diabetic diet. The patient states, "What difference does it make if you drink the juice or eat the fruit? It is all the same." What is the best response by the nurse?

"Eating the fruit instead of drinking juice decreases the glycemic index by slowing absorption." Eating whole fruit instead of drinking juice decreases the glycemic index, because fiber in the fruit slows absorption.

Which statement indicates that a client with diabetes mellitus understands proper foot care?

"I'll wear cotton socks with well-fitting shoes."

A nurse is assessing a client who has systemic lupus erythematosus (SLE). Which of the following findings should the nurse expect? (Select all that apply) -Subcutaneous nodules -Decreased urine output -Renal calculi -Butterfly rash -Joint inflammation

-decreased urine output -butterfly rash -joint inflammation

A client with diabetes mellitus has a blood glucose level of 40 mg/dL. Which rapidly absorbed carbohydrate would be most effective?

1/2 cup fruit juice or regular soft drink

What is the duration of regular insulin?

4 to 6 hours

A client with diabetes comes to the clinic for a follow-up visit. The nurse reviews the client's glycosylated hemoglobin test results. Which result would indicate to the nurse that the client's blood glucose level has been well-controlled?

6.5%

The nurse is aware that the best time of day for the total large corticosteroid dose is between:

7:00 AM and 8:00 AM The best time of day for the total large corticosteroid dose is in the early morning, between 7:00 AM and 8:00 AM, when the adrenal gland is most active. Therefore, dosage at this time of day will result in the maximum suppression of the adrenal gland.

A nurse is monitoring a client's status 24hr after a total thyroidectomy. Which of the following findings should the nurse report to the provider? A. Laryngeal Stridor. B. Productive cough C. Pain with hyperextension of the neck. D. Hoarse, weak voice

A.

A nurse is performing an assessment on a client who has SIADH. Which of the following assessment data should the nurse report? A. Serum sodium 110 mEq/L B. 2+ DTRs C. Serum K+ 3.7 mEq/L D. Urine specific gravity 1.025

A.

After receiving a dose of penicillin, a client develops dyspnea and hypotension. The nurse suspects the client is experiencing anaphylactic shock. What should the nurse do first?

Administer epinephrine, as ordered, and prepare to intubate the client, if necessary.

The nurse is evaluating a client's readiness for allergy skin testing. The nurse determines that the testing will need to be postponed when it is revealed that the client took which classification of medication the night before?

Antihistamine

Which of the following procedures involves a surgical fusion of the joint?

Arthrodesis

What is the priority intervention for a client who has been admitted repeatedly with attacks of gout?

Assess diet and activity at home

Which of the following cell types are involved in humoral immunity?

B lymphocytes

nsulin is secreted by which of the following types of cells?

Beta cells

A nurse is caring for a client who is admitted with enlarged lymph nodes and a fever. To confirm a diagnosis of bacterial pharyngitis, the nurse should anticipate which of the following diagnostic tests? A. Indirect laryngoscopy B. chest x ray C. throat culture D. monospot test

C.

A nurse is caring for a client who is taking propylthiouracil (PTU). The nurse should recognize that the client has met the treatment goals when she reports an increase in which of the following effects? A. Sweating B. Stools C. Weight D. Appetite

C.

A nurse is prepping insulin for a client with DM. He is to receive evening doses of insulin glargine and regular insulin. Which of the following actions should the nurse take to administer these two medications safely? A. Draw up the insulin glargine into the syringe first, then the regular insulin. B. Draw up the regular insulin first, then the insulin glargine C. Draw up the insulin glargine and the regular insulin into two separate syringes. D. Draw up either insulin into the syringe first because both are clear.

C.

Trousseau's sign is elicited by which of the following?

Carpopedal spasm is induced by occluding the blood flow to the arm for 3 minutes with the use of a blood pressure cuff. A positive Trousseau's sign is suggestive of latent tetany. A positive Chvostek's sign is demonstrated when a sharp tapping over the facial nerve just in front of the parotid gland and anterior to the ear causes spasm or twitching of the mouth, nose, and eye. A positive Allen's test is demonstrated by the palm remaining blanched with the radial artery occluded. A positive Homans' sign is demonstrated when the patient complains of pain in the calf when his foot is dorsiflexed.

A client is diagnosed with diabetes mellitus. Which assessment finding best supports a nursing diagnosis of Ineffective coping related to diabetes mellitus?

Crying whenever diabetes is mentioned

The nurse is caring for a client with myasthenia gravis. The nurse generates a plan of care for the client based on which type of hypersensitivity reaction?

Cytotoxic

This type of T lymphocyte is responsible for altering the cell membrane and initiating cellular lysis. Choose the T lymphocyte.

Cytotoxic T cell

The nurse is obtaining information from a client with Crohn's disease about his medication history. What medication would the nurse include when asking about what medications the client has taken for suppression of the inflammatory and immune response?

Corticosteroids

A client comes to the emergency department complaining of pain in the right leg. When obtaining the history, the nurse learns that the client has a history of obesity and hypertension. Based on this information the nurse anticipates the client having which musculoskeletal disorder?

Degenerative joint disease

A client calls the clinic and asks the nurse if using oxymetazoline nasal spray would be alright to relieve the nasal congestion he is experiencing due to seasonal allergies. What instructions should the nurse provide to the client to avoid complications?

Do not overuse the medication as rebound congestion can occur.

When assessing the skin of a client with allergic contact dermatitis, the nurse would most likley expect to find irritation at which area?

Dorsal aspect of the hand

The nurse is caring for a female patient who has an exacerbation of lupus erythematosus. What does the nurse understand is the reason that females tend to develop autoimmune disorders more frequently than men?

Estrogen tends to enhance immunity.

Which of the following would the nurse expect to find in a client with severe hyperthyroidism?

Exophthalmos Exophthalmos that results from enlarged muscle and fatty tissue surrounding the rear and sides of the eyeball is seen in clients with severe hyperthyroidism. Tetany is the symptom of acute and sudden hypoparathyroidism. Buffalo hump and striae are the symptoms of Cushing's syndrome.

A nurse should perform which intervention for a client with Cushing's syndrome?

Explain that the client's physical changes are a result of excessive corticosteroids. The nurse should explain to the client that Cushing's syndrome causes physical changes related to excessive corticosteroids. Clients with hyperthyroidism, not Cushing's syndrome, are heat intolerant and must have cool clothing and bedding. Clients with Cushing's syndrome should have a high-protein, not low-protein, diet. Clients with Addison's disease must increase sodium intake and fluid intake in times of stress of prevent hypotension.

A client with lupus has had antineoplastic drugs prescribed. Why would the physician prescribe antineoplastic drugs for an autoimmune disorder?

For their immunosuppressant effects

Which feature(s) indicates a carpopedal spasm in a client with hypoparathyroidism?

Hand flexing inward Carpopedal spasm is evidenced by the hand flexing inward. Cardiac dysrhythmia is a symptom of hyperparathyroidism. Moon face and buffalo hump are the symptoms of Cushing syndrome. A bulging forehead is a symptom of acromegaly.

Which term refers to an incomplete antigen?

Hapten

When obtaining a health history from a patient with possible abnormal immune function, what question would be a priority for the nurse to ask?

Have you ever received a blood transfusion?"

On the third day after a partial thyroidectomy, a client exhibits muscle twitching and hyperirritability of the nervous system. When questioned, the client reports numbness and tingling of the mouth and fingertips. Suspecting a life-threatening electrolyte disturbance, the nurse notifies the surgeon immediately. Which electrolyte disturbance most commonly follows thyroid surgery?

Hypocalcemia Hypocalcemia may follow thyroid surgery if the parathyroid glands were removed accidentally. Signs and symptoms of hypocalcemia may be delayed for up to 7 days after surgery. Thyroid surgery doesn't directly cause serum sodium, potassium, or magnesium abnormalities. Hyponatremia may occur if the client inadvertently received too much fluid; however, this can happen to any surgical client receiving I.V. fluid therapy, not just one recovering from thyroid surgery. Hyperkalemia and hypermagnesemia usually are associated with reduced renal excretion of potassium and magnesium, not thyroid surgery.

A client with diabetes mellitus is receiving an oral antidiabetic agent. The nurse observes for which symptom when caring for this client?

Hypoglycemia

When developing a care plan for a client newly diagnosed with scleroderma, which nursing diagnosis has the highest priority?

Impaired skin integrity

The nurse is educating the client with diabetes on setting up a sick plan to manage blood glucose control during times of minor illness such as influenza. Which is the most important teaching item to include?

Increase frequency of glucose self-monitoring. The body's need for insulin increases during illness.

A 60-year-old client comes to the ED reporting weakness, vision problems, increased thirst, increased urination, and frequent infections that do not seem to heal easily. The physician suspects that the client has diabetes. Which classic symptom should the nurse watch for to confirm the diagnosis of diabetes?

Increased hunger The classic symptoms of diabetes are the three Ps: polyuria (increased urination), polydipsia (increased thirst), and polyphagia (increased hunger).

A client newly diagnosed with diabetes mellitus asks why he needs ketone testing when the disease affects his blood glucose levels. How should the nurse respond?

Ketones will tell us if your body is using other tissues for energy The nurse should tell the client that ketones are a byproduct of fat metabolism and that ketone testing can determine whether the body is breaking down fat to use for energy. The spleen doesn't release ketones when the body can't use glucose. Although ketones can damage the eyes and kidneys and help the physician evaluate the severity of a client's diabetes, these responses by the nurse are incomplete.

Which type of insulin acts most quickly?

Lispro

he nurse is educating the patient with diabetes about the importance of increasing dietary fiber. What should the nurse explain is the rationale for the increase? Select all that apply.

May improve blood glucose levels Decrease the need for exogenous insulin Help reduce cholesterol levels

Which joint is most commonly affected in gout?

Metatarsophalangeal

A client with diabetes is receiving an oral antidiabetic agent that acts to help the tissues use available insulin more efficiently. Which of the following agents would the nurse expect to administer?

Metformin

A nurse is reviewing the dietary history of a client who has experienced anaphylaxis. Which of the following would the nurse identify as a common cause of anaplhylaxis? Select all that apply?

Milk Eggs Shrimp

The clinic nurse is caring for a client with an allergic disorder who has received the first sensitizing dose of a new drug. Now, the client is given a second dose. What nursing action is most important at this stage of transition?

Monitor the client for reactions.

A patient has had a "stuffy nose" and obtained Afrin nasal spray. What education should the nurse provide to the patient in order to prevent "rebound congestion"?

Only use the Afrin for 3 to 4 days once every 12 hours.

Which of the following is the most common cause for a patient to seek medical attention for arthritis?

Pain

Which action would be appropriate when evaluating a client's neck for thyroid enlargement?

Palpate the thyroid gland gently. The nurse should inspect the neck for thyroid enlargement and gently palpate the thyroid gland. Repeated palpation of the thyroid in case of thyroid hyperactivity can result in a sudden release of a large amount of thyroid hormones, which may have serious implications. Pigment changes in the neck and excessive oiliness of the skin are not related to assessment for thyroid enlargement.

The nurse is caring for a client with hypoparathyroidism. When the nurse taps the client's facial nerve, the client's mouth twitches and the jaw tightens. What is this response documented as related to the low calcium levels?

Positive Chvostek's sign If a nurse taps the client's facial nerve (which lies under the tissue in front of the ear), the client's mouth twitches and the jaw tightens. The response is identified as a positive Chvostek's sign. A positive Trousseau's sign is elicited by placing a BP cuff on the upper arm, inflating it between the systolic and diastolic BP, and waiting 3 minutes. The nurse observes the client for spasm of the hand (carpopedal spasm), which is evidenced by the hand flexing inward. Positive Babinski's sign is elicited by stroking the sole of the foot. Paresthesia is not a symptom that can be elicited; it is felt by the client.

A client is diagnosed with multiple site cancers and has received whole-body irradiation. The nurse is concerned about a compromised immune system in this client for which of the following reasons?

Radiation destroys lymphocytes.

Lispro (Humalog) is an example of which type of insulin?

Rapid-acting

The result of which diagnostic study is decreased in the client diagnosed with rheumatoid arthritis?

Red blood cell count

The nurse is performing discharge teaching for a client with rheumatoid arthritis. What teachings are priorities for the client? Select all that apply.

Safe exercise Medication dosages and side effects Assistive devices

The diabetic client asks the nurse why shoes and socks are removed at each office visit. Which assessment finding is most significant in determining the protocol for inspection of feet?

Sensory neuropathy Neuropathy results from poor glucose control and decreased circulation to nerve tissues.

A nurse is taking the health history of a newly admitted client. Which of the following conditions would NOT place the client at risk for impaired immune function?

Surgical removal of the appendix

The nurse is gathering objective data for a client at the clinic complaining of arthritic pain in the hands. The nurse observes that the fingers are hyperextended at the proximal interphalangeal joint with fixed flexion of the distal interphalangeal joint. What does the nurse recognize this deformity as?

Swan neck deformity

The nurse is beginning the physical examination of a client with a complaint of fatigue. What documentation will the nurse provide to describe this general appraisal of the client's health?

The client appears mildly ill, listless, and disheveled.

Which of the following factors would a nurse identify as a most likely cause of diabetic ketoacidosis (DKA) in a client with diabetes?

The client has eaten and has not taken or received insulin.

Which may be a potential cause of hypoglycemia in the client diagnosed with diabetes mellitus?

The client has not eaten but continues to take insulin or oral antidiabetic medications.

A nurse is preparing to administer two types of insulin to a client with diabetes mellitus. What is the correct procedure for preparing this medication?

The short-acting insulin is withdrawn before the intermediate-acting insulin. When combining two types of insulin in the same syringe, the short-acting regular insulin is withdrawn into the syringe first and the intermediate-acting insulin is added next. This practice is referred to as "clear to cloudy."

A client is scheduled to have a prick test to determine what specific allergens are creating problems for the client. What should the nurse inform the client is involved with the testing?

The skin will be scratched, and applying a small amount of the liquid test antigen to the scratch, usually on the back.

Which statement accurately reflects current stem cell research?

The stem cell is known as a precursor cell that continually replenishes the body's entire supply of both red and white cells.

T-cell deficiency occurs when which gland fails to develop normally during embryogenesis?

Thymus

Which intervention is essential when performing dressing changes on a client with a diabetic foot ulcer?

Using sterile technique during the dressing change

What test will the nurse assess to determine the client's response to antiretroviral therapy?

Viral load

A nurse is managing the care of a client with osteoarthritis. Appropriate treatment strategies for osteoarthritis include:

administration of nonsteroidal anti-inflammatory drugs (NSAIDs) and initiation of an exercise program.

A client is being treated for hyperuricemia. Part of the treatment strategy is for the client to avoid contributing factors whenever possible. Which activities might bring on an acute attack?

eating organ meats and sardines

A client has been experiencing a decrease in serum calcium. After diagnostics, the physician proposes the calcium level fluctuation is due to altered parathyroid function. What is the typical number of parathyroid glands?

four The parathyroid glands are four (some people have more than four) small, bean-shaped bodies, each surrounded by a capsule of connective tissue and embedded within the lateral lobes of the thyroid.

A client tells the nurse that she has been working hard for the past 3 months to control her type 2 diabetes with diet and exercise. To determine the effectiveness of the client's efforts, the nurse should check:

glycosylated hemoglobin level. Because some of the glucose in the bloodstream attaches to some of the hemoglobin and stays attached during the 120-day life span of red blood cells, glycosylated hemoglobin levels provide information about blood glucose levels during the previous 3 months.

The body has several mechanisms to fight disease, one of which is sending chemical messengers. The messengers released by lymphocytes, monocytes, and macrophages have differing roles in the immune response. Which messenger enables cells to resist viral replication and slow viral replication?

interferons

The body has several mechanisms to fight disease, one of which is sending chemical messengers. The messengers released by lymphocytes, monocytes, and macrophages have differing roles in the immune response. Which messenger promotes inflammation, fever, and angiogenesis?

interleukins

A client is undergoing diagnostics for an alteration in thyroid function. What physiologic function is affected by altered thyroid function?

metabolic rate The thyroid concentrates iodine from food and uses it to synthesize thyroxine (T4) and triiodothyronine (T3). These two hormones regulate the body's metabolic rate.

The nurse is talking with a group of teens about transmission of human immunodeficiency virus (HIV). What body fluids does the nurse inform them will transmit the virus? Select all that apply.

semen breast milk blood vaginal secretions

A client with Addison's disease comes to the clinic for a follow-up visit. When assessing this client, the nurse should stay alert for signs and symptoms of:

sodium and potassium abnormalities. In Addison's disease, a form of adrenocortical hypofunction, aldosterone secretion is reduced. Aldosterone promotes sodium conservation and potassium excretion. Therefore, aldosterone deficiency increases sodium excretion, predisposing the client to hyponatremia, and inhibits potassium excretion, predisposing the client to hyperkalemia. Because aldosterone doesn't regulate calcium, phosphorus, chloride, or magnesium, an aldosterone deficiency doesn't affect levels of these electrolytes directly.

The lower the client's viral load

the longer the survival time.

A nurse is caring for a client with diabetes insipidus. The nurse should anticipate administering:

vasopressin Vasopressin is given subcutaneously to manage diabetes insipidus. Insulin is used to manage diabetes mellitus. Furosemide causes diuresis. Potassium chloride is given for hypokalemia.

A client is recovering from an attack of gout. Client teaching should include the need to lose weight because:

weight loss will reduce uric acid levels and reduce stress on joints.

Which of the following are modes of transmission for HIV? Select all that apply.

Blood Seminal fluid Vaginal secretions Amniotic fluid

Which clinical characteristic is associated with type 2 diabetes (previously referred to as non-insulin-dependent diabetes mellitus)?

Blood glucose can be controlled through diet and exercise

Which clinical manifestation of type 2 diabetes occurs if glucose levels are very high?

Blurred vision

The nurse is teaching a client about rheumatic disease. What statement best helps to explain autoimmunity?

"Your symptoms are a result of your body attacking itself."

A nurse should expect to administer which medication to a client with gout?

Colchicine

The nurse is preparing to administer intermediate-acting insulin to a patient with diabetes. Which insulin will the nurse administer?

NPH

Preventive health education includes informing patients about the symptoms associated with allergic rhinitis. The nurse is aware that the most common trigger to this respiratory allergic response is:

Plant pollen.

Which of the following is the definitive diagnostic of gouty arthritis?

Polarized light microscopy of the synovial fluid

A patient with a history of allergies comes to the clinic for an evaluation. The following laboratory test findings are recorded in a patient's medical record: Total serum IgE levels: 2.8 mg/mL White blood cell count: 5,100/cu mm Eosinophil count: 4% Erythrocyte sedimentation rate: 20 mm/h The nurse identifies which result as suggesting an allergic reaction?

Serum IgE level

A nurse explains to a client with thyroid disease that the thyroid gland normally produces:

T3, thyroxine (T4), and calcitonin. The thyroid gland normally produces thyroid hormone (T3 and T4) and calcitonin. The pituitary gland produces TSH to regulate the thyroid gland. The hypothalamus gland produces TRH to regulate the pituitary gland.

The nurse is teaching a client about self-administration of insulin and about mixing regular and neutral protamine Hagedorn (NPH) insulin. Which information is important to include in the teaching plan?

When mixing insulin, the regular insulin is drawn up into the syringe first.

The presence of crystals in synovial fluid obtained from arthrocentesis is indicative of

gout.

The best treatment for latex allergy is:

avoidance of latex-based products.

A 20-year-old client cut a hand while replacing a window. While reviewing the complete blood count (CBC) with differential, the nurse would expect which cell type to be elevated first in an attempt to prevent infection in the client's hand?

Neutrophils

Which leukocytes arrive at the inflammation site first?

Neutrophils

Which medication classification is known to inhibit prostaglandin synthesis or release?

Nonsteroidal anti-inflammatory drugs (in large doses)

When could a "rebound" anaphylactic reaction occur after an initial attack even when epinephrine has been given?

4 hours

Which is usually the most important consideration in the decision to initiate antiretroviral therapy?

CD4+ counts

A nurse knows that more than 50% of clients with CVID develop the following disorder.

Pernicious anemia

A young client has a significant height deficit and is to be evaluated for diagnostic purposes. What could be the cause of this client's disorder?

pituitary disorder Pituitary disorders usually result from excessive or deficient production and secretion of a specific hormone. Dwarfism occurs when secretion of growth hormone is insufficient during childhood.

A client has been recently diagnosed with type 2 diabetes, and reports continued weight loss despite increased hunger and food consumption. This condition is called:

polyphagia. The person with diabetes feels hungry and eats more (polyphagia). Despite eating more, he or she loses weight as the body uses fat and protein to substitute for glucose.

A nurse is caring for a client with human immunodeficiency virus (HIV). To determine the effectiveness of treatment the nurse expects the physician to order:

quantification of T-lymphocytes.

The stage of immune response that uses lymph node and lymphocytes for surveillance is the

recognition stage

Which type of immunity becomes active as a result of infection by a specific microorganism?

Naturally acquired active immunity

A patient who is diagnosed with type 1 diabetes would be expected to:

Need exogenous insulin. Type 1 diabetes is characterized by the destruction of pancreatic beta cells that require exogenous insulin.

A nurse expects to find which signs and symptoms in a client experiencing hypoglycemia?

Nervousness, diaphoresis, and confusion

Which glands regulate calcium and phosphorous metabolism?

Parathyroid Parathormone (parathyroid hormone), the protein hormone produced by the parathyroid glands, regulates calcium and phosphorous metabolism. The thyroid gland controls cellular metabolic activity. The adrenal medulla at the center of the adrenal gland secretes catecholamines, and the outer portion of the gland, the adrenal cortex, secretes steroid hormones. The pituitary gland secretes hormones that control the secretion of additional hormones by other endocrine glands.

A laboring mother asks the nurse if the baby will have immunity to some illnesses when born. What type of immunity does the nurse understand that the newborn will have?

Passive immunity transferred by the mother

The nurse working the medical-surgical floor knows that the nitroblue tetrazolium reductase (NTR) test is used to diagnose immunodeficiency disorders related to

Phagocytic cells

Which type of contact dermatitis requires light exposure in addition to allergen contact?

Photoallergic

What intervention is a priority for a client diagnosed with osteoarthritis?

Physical therapy and exercise

A client suspected of having HIV has blood drawn for a screening test. What is the first test generally run to see if a client is, indeed, HIV positive?

ELISA

A client in a late stage of acquired immunodeficiency syndrome (AIDS) shows signs of AIDS-related dementia. Which nursing diagnosis takes highest priority?

Risk for injury

A 1200-calorie diet and exercise are prescribed for a client with newly diagnosed type 2 diabetes. The nurse is teaching the client about meal planning using exchange lists. The teaching is determined to be effective based on which statement by the client?

"For dinner I ate a 3-ounce hamburger on a bun, with ketchup, pickle, and onion; a green salad with 1 teaspoon Italian dressing; 1 cup of watermelon; and a diet soda." There are six main exchange lists: bread/starch, vegetable, milk, meat, fruit, and fat. Foods within one group (in the portion amounts specified) contain equal numbers of calories and are approximately equal in grams of protein, fat, and carbohydrate.

The nurse is caring for a young client who has agammaglobulinemia. The nurse is teaching the family how to avoid infection at home. Which statement by the family indicates that additional teaching is needed?

"I can take my child to the beach, as long as we play in the sand rather than swim in the water."

After teaching a client with type 1 diabetes who is scheduled to undergo an islet cell transplant, which client statement indicates successful teaching?

"I might need insulin later on but probably not as much or as often."

The nurse is teaching the client who has an immunodeficiency disorder how to avoid infection at home. Which statement indicates that additional teaching is needed?

"I will be sure to eat lots of fresh fruits and vegetables every day."

A nurse is caring for a client who was recently diagnosed with hyperparathyroidism. Which statement by the client indicates the need for additional discharge teaching?

"I will increase my fluid and calcium intake." The client requires additional teaching if he states that he will increase his calcium intake. Hyperparathyroidism causes extreme increases in serum calcium levels. The client should increase his fluid intake, but he should limit his calcium and vitamin D intake. The client should continue to take pain mediations as scheduled and have regular follow-up visits with his physician. Tingling around the lips is a sign of hypercalcemia and should be reported to the physician immediately.

A client is evaluated for type 1 diabetes. Which client comment correlates best with this disorder?

"I'm thirsty all the time. I just can't get enough to drink."

When evaluating a client's knowledge about use of antihistamines, which of the following statements made by the patient would indicate to the nurse a knowledge deficit?

"If I am pregnant, I should take half the dose."

A client presents at the clinic with an allergic disorder. The client asks the nurse what an "allergic disorder" means. What would be the nurse's best response?

"It is a hyperimmune response to something in the environment that is usually harmless."

A nurse is teaching a client recovering from diabetic ketoacidosis (DKA) about management of "sick days." The client asks the nurse why it is important to monitor the urine for ketones. Which statement is the nurse's best response?

"Ketones accumulate in the blood and urine when fat breaks down in the absence of insulin. Ketones signal an insulin deficiency that will cause the body to start breaking down stored fat for energy."

A nurse is caring for client who has a new prescirption for clindamycin to treat acute pelvic inflammatory disease. The nurse should monitor for and report which of the following finding immediately to the provider? A. Watery diarrhea B. Vaginitis C. fever D. N/V

A. greatest risk is pseudomembranous colitis, immediately discontinue the medicstion

A nurse is teaching a client recovering from diabetic ketoacidosis (DKA) about management of "sick days." The client asks the nurse why it is important to monitor the urine for ketones. Which statement is the nurse's best response?

"Ketones accumulate in the blood and urine when fat breaks down in the absence of insulin. Ketones signal an insulin deficiency that will cause the body to start breaking down stored fat for energy." Ketones (or ketone bodies) are by-products of fat breakdown in the absence of insulin, and they accumulate in the blood and urine. Ketones in the urine signal an insulin deficiency and that control of type 1 diabetes is deteriorating. When almost no effective insulin is available, the body starts to break down stored fat for energy.

A client with rheumatoid arthritis arrives at the clinic for a checkup. Which statement by the client refers to the most overt clinical manifestation of rheumatoid arthritis?

"My finger joints are oddly shaped."

A client with osteoarthritis tells the nurse she is concerned that the disease will prevent her from doing her chores. Which suggestion should the nurse offer?

"Pace yourself and rest frequently, especially after activities."

A patient tells the nurse, "I can't believe I have ineffective immune function and am getting sick again. I exercise rigorously and compete regularly." What is the best response by the nurse?

"Rigorous exercise can cause negative effects on immune response."

A client with diabetes mellitus must learn how to self-administer insulin. The physician has ordered 10 units of U-100 regular insulin and 35 units of U-100 isophane insulin suspension (NPH) to be taken before breakfast. When teaching the client how to select and rotate insulin injection sites, the nurse should provide which instruction?

"Rotate injection sites within the same anatomic region, not among different regions." Rotating sites among different regions may cause excessive day-to-day variations in the blood glucose level; also, insulin absorption differs from one region to the next.

A nurse is explaining treatment options to a client diagnosed with an immune dysfunction. Which statement by the client accurately reflects the teaching about current stem cell research?

"Stem cell transplantation has been carried out in humans with certain types of immune dysfunction, and clinical trials using stem cells are underway in clients with a variety of disorders with an autoimmune component."

A client suspected of having systemic lupus erythematosus (SLE) is being scheduled for testing. She asks which of the tests ordered will determine if she is positive for the disorder. Which statement by the nurse is most accurate?

"The diagnosis won't be based on the findings of a single test but by combining all data found."

A nurse is teaching the parents of an infant about primary immunodeficiencies. Which statement verifies that the parents understand the teaching?

"The majority of primary immunodeficiencies are diagnosed in infancy."

A client with gout has been advised to lose weight. She informs the nurse that she plans to go on a "guaranteed rapid weight loss" plan that involves fasting and heavy exercise. Which response would be most appropriate?

"There might be some difficulties with your plan and fasting."

A parent of a child who has been having frequent bouts of tonsillitis brings the child back to the clinic for another sore throat. The parent asks the nurse, "What are tonsils good for anyway?" What is the best response by the nurse?

"These tissues filter bacteria from tissue fluid."

The nurse is obtaining an assessment and health history from the parents of a 6-month-old infant with an elevated temperature. Which statement by the parents alerts the nurse to a possible immunodeficiency disorder?

"This is the third infection with a high fever the baby has had in the past month."

Health teaching for a patient with diabetes who is prescribed Humulin N, an intermediate NPH insulin, would include which of the following advice?

"You should take your insulin after you eat breakfast and dinner."

A client has just been diagnosed with type 1 diabetes. When teaching the client and family how diet and exercise affect insulin requirements, the nurse should include which guideline?

"You'll need less insulin when you exercise or reduce your food intake." he nurse should advise the client that exercise, reduced food intake, hypothyroidism, and certain medications decrease insulin requirements. Growth, pregnancy, greater food intake, stress, surgery, infection, illness, increased insulin antibodies, and certain medications increase insulin requirements.

A 16-year-old client newly diagnosed with type 1 diabetes has a very low body weight despite eating regular meals. The client is upset because friends frequently state, "You look anorexic." Which statement by the nurse would be the best response to help this client understand the cause of weight loss due to this condition?

"Your body is using protein and fat for energy instead of glucose."

A physician orders blood glucose levels every 4 hours for a 4-year-old child with brittle type 1 diabetes. The parents are worried that drawing so much blood will traumatize their child. How can the nurse best reassure the parents?

"Your child will need less blood work as his glucose levels stabilize."

There are major differences between primary and secondary immunodeficiencies. Select the most accurate statement the nurse would use to explain the cause of a secondary immunodeficiency.

"Your immune system was most likely affected by an underlying disease process."

Which are correct statements about the relationship between the hypothalamus and the pituitary gland? Select all that apply.

-Many endocrine glands respond to stimulation from the pituitary gland, which is connected by a stalk to the hypothalamus in the brain. -Under the influence of the hypothalamus, the lobes of the pituitary gland secrete various hormones. Many endocrine glands respond to stimulation from the pituitary gland, which is connected by a stalk to the hypothalamus in the brain. Under the influence of the hypothalamus, the lobes of the pituitary gland secrete various hormones. Even though the pituitary gland is called the 'master gland,' the hypothalamus influences the pituitary gland. The pituitary gland is called the 'master gland' because it regulates the function of other endocrine glands.

An agitated, confused client arrives in the emergency department. The client's history includes type 1 diabetes, hypertension, and angina pectoris. Assessment reveals pallor, diaphoresis, headache, and intense hunger. A stat blood glucose sample measures 42 mg/dl, and the client is treated for an acute hypoglycemic reaction. After recovery, the nurse teaches the client to treat hypoglycemia by ingesting:

10 to 15 g of a simple carbohydrate. o reverse hypoglycemia, the American Diabetes Association recommends ingesting 10 to 15 g of a simple carbohydrate, such as three to five pieces of hard candy, two to three packets of sugar (4 to 6 tsp), or 4 oz of fruit juice.

The nurse is administering lispro insulin. Based on the onset of action, how long before breakfast should the nurse administer the injection?

10 to 15 minutes

Patient teaching about mild allergic responses should include information about the onset of symptoms. The nurse tells the patient that if exposure to an allergen occurs around 8:00 AM, he should expect a mild or moderate reaction by what time?

10:00 AM

A patient who is 6 months' pregnant was evaluated for gestational diabetes mellitus. The doctor considered prescribing insulin based on the serum glucose result of:

138 mg/dL, 2 hours postprandial. The goals for a 2-hour, postprandial blood glucose level are less than 120 mg/dL in a patient who might develop gestational diabetes.

A nurse is teaching a client with diabetes mellitus about self-management of his condition. The nurse should instruct the client to administer 1 unit of insulin for every:

15 g of carbohydrates.

A nurse is caring for a client with type 1 diabetes who exhibits confusion, light-headedness, and aberrant behavior. The client is conscious. The nurse should first administer:

15 to 20 g of a fast-acting carbohydrate such as orange juice.

While monitoring the patient's eosinophil level, the nurse suspects a definite allergic disorder when seeing an eosinophil value of what percentage of the total leukocyte count?

15% to 40%

A patient comes to the clinic with pruritus and nasal congestion after eating shrimp for lunch. The nurse is aware that the patient may be having an anaphylactic reaction to the shrimp. These symptoms typically occur within how many hours after exposure?

2 hours

Glycosylated hemoglobin reflects blood glucose concentrations over which period of time?

3 months

A patient is on highly active antiretroviral therapy (HAART) for the treatment of HIV. What does the nurse know would be an adequate CD4 count to determine the effectiveness of treatment for a patient per year?

50 mm3 to 150 mm3

A client is beginning highly active antiretroviral therapy (HAART). The client demonstrates an understanding of the need for follow up when he schedules a return visit for viral load testing at which time?

6 weeks

A client is receiving insulin lispro at 7:30 AM. The nurse ensures that the client has breakfast by which time?

7:45 AM Insulin lispro has an onset of 5 to 15 minutes. Therefore, the nurse would need to ensure that the client has his breakfast by 7:45 AM at the latest. Otherwise, the client may experience hypoglycemia.

A health care provider prescribes short-acting insulin for a patient, instructing the patient to take the insulin 20 to 30 minutes before a meal. The nurse explains to the patient that Humulin-R taken at 6:30 AM will reach peak effectiveness by:

8:30am

The nurse is instructing clients about the importance of taking the shingles vaccine. Which client would benefit from this vaccine?

A 65-year-old client who had chicken pox when he was 12 years old

A patient is ordered desmopressin (DDAVP) for the treatment of diabetes insipidus. What therapeutic response does the nurse anticipate the patient will experience?

A decrease in urine output Desmopressin (DDAVP), a synthetic vasopressin without the vascular effects of natural ADH, is particularly valuable because it has a longer duration of action and fewer adverse effects than other preparations previously used to treat the disease. DDAVP and lypressin (Diapid) reduce urine output to 2 to 3 L/24 hours. It is administered intranasally; the patient sprays the solution into the nose through a flexible calibrated plastic tube. One or two administrations daily (i.e., every 12 to 24 hours) usually control the symptoms (Papadakis, McPhee, & Rabow, 2013). Vasopressin causes vasoconstriction; thus, it must be used cautiously in patients with coronary artery disease.

Which statement is true regarding gestational diabetes?

A glucose challenge test should be performed between 24 to 28 weeks.

A nurse is providing teaching for a client who has a new prescription of amoxicillin to treat a respiratory infection. Which of the following statements by the client indicates an understanding of these teachings? A. "My birth control pills are less effective while I am on this medication" B. " I must take this medication on an empty stomach" C. "I should expect to have constipation while taking this medication" D. "I will keep taking this medication until I feel better"

A. antibiotics accelerate the elimination of oral contraceptives, making them less effective

A nurse is teaching a client who has an autoimmune disease about the adverse effects of long-term corticosteroid therapy. Which of the following effects should the nurse include? (Select All) A Osteoporosis B Moon-shaped face C Increased risk of infection D Hearing loss E Weight loss

A, B, C

A nurse id providing teaching for a group of clients regarding prevention of skin cancer. Which of the following risk factors should the nurse include in the teaching? A. Light skin pigmentation B. Psoriasis C. history of frostbite D. Immunodeficiency disorder

A.

A nurse is caring for a client who has a pheochromocytoma. Which of the following actions should the nurse take? A. Elevate the head of the client's bed B. Palpate the client's abdomen C. Monitor the client for hypotension D. Check the client's urine specific gravity

A.

A nurse is developing a teaching plan for a client who had a thyroidectomy and takes a thyroid hormone replacement. Which of the following instructions should the nurse plan to include? A. Take this med on an empty stomach B Take this med with an antacid C. Change position slowly while taking this med D. Limit your fluid intake while taking this med

A.

A nurse is managing the care of a client who is post-op and experiencing acute adrenal insufficiency. Which of the following actions should the nurse take? A. Administer IV hydrocortisone sodium succinate. B. Give oral spironolactone. C. Infuse 1 unit of platelets. D. Restrict daily fluid intake.

A.

A nurse is monitoring the lab values of a client who has DM and is taking insulin. Which of the following results indicates a therapeutic outcome of insulin therapy? A. Fasting blood glucose 96 mg/dL B. Postprandial blood glucose 195 mg/dL C. Casual blood glucose 210 mg/dL D. Preprandial blood glucose 60 mg/dL

A.

A nurse is preparing to administer propranolol IV bolus to a client experiencing a thyroid storm. Which of the following findings indicates the client is having a therapeutic response? A. Reduction of the effects of thyroid hormone on the heart B. Blockage of the release of thyroid hormone from the thyroid gland. C. Increase of the heart's sensitivity to thyroid hormone. D. Increase of the uptake of thyroid hormone by the thyroid gland.

A.

A nurse is reviewing laboratory values for a client who has diabetic ketoacidosis (DKA). Which of the following results should the nurse expect? A. pH 7.32, PaCO2 36 mm Hg, HCO3- 14 mEq/L B. pH 7.38, PaCO2 55 mm Hg, HCO3- 22 mEq/L C. pH 7.44, PaCO2 40 mm Hg, HCO3- 24 mEq/L D. pH 7.50, PaCO2 42 mm Hg, HCO3- 30 mEq/L

A.

A nurse is providing teaching for a client who has rheumatoid arthritis and a new prescription for methotrexate. Which of the following should the nurse include in the teaching? A. Avoid crowds B. Expect symptoms to subside in 1 to 2 weeks C. Increase intake of vitamin D D. Anticipate constipation

A. can decrease WBC and platelet levels, thus increasing risk for infection

A nurse is caring for a client who reports a skin change on her arm. Which of the following findings should the nurse report to the provider? A. An asymmetrical papule that is pigmented B. A patch of silvery-white scales with a red epidermal base C. A collection of irregular dry papules that are black D. An elevated red lesion that arises from a scar

A. indicates malignant melanoma

A nurse is assessing a client who has HIV. Which of the following findings should cause the nurse to suspect that the client's diagnosis has progressed to AIDS? A. Small purple-colored skin lesions B. Fever and diarrhea lasting longer than 1 month C. Persistent, generalized lymphadenopathy D. CD4 T-cells decreased to 750

A. means acquired Kaposi's sarcoma, which is an AIDS-defining illness.

A nurse is caring for a client who has neutropenia. Which of the following findings indicates a need for intervention? A. The client's granddaughter is visiting and telling him about her first day of kindergarten. B. The client has a grilled ham and cheese sandwich, a banana, and yogurt on his lunch tray C. The client's family brings in a silk flower arrangement D. the client's assistive personnel places paper cups and plastic utensils in his room.

A. no no he gas immunocompromised status

A nurse is reviewing the daily laboratory results for a female client who has acute leukemia. Which of the following values is an expected finding? A. WBC 21,00 B. hgb 14 C. hct 40% D. Platelets 170,00

A. typically have an elevated WBC

A nurse educator is preparing to discuss immunodeficiency disorders with a group of fellow nurses. What would the nurse identify as the most common secondary immunodeficiency disorder?

AIDS

When administering insulin to a client with type 1 diabetes, which of the following would be most important for the nurse to keep in mind?

Accuracy of the dosage

A client with type 1 diabetes must undergo bowel resection in the morning. How should the nurse proceed while caring for him on the morning of surgery?

Administer half of the client's typical morning insulin dose as ordered. If the nurse administers the client's normal daily dose of insulin while he's on nothing-by-mouth status before surgery, he'll experience hypoglycemia.

Before discharge, what should a nurse instruct a client with Addison's disease to do when exposed to periods of stress?

Administer hydrocortisone I.M. Clients with Addison's disease and their family members should know how to administer I.M. hydrocortisone during periods of stress. Although it's important for the client to keep well hydrated during stress, the critical component in this situation is to know how and when to use I.M. hydrocortisone. Capillary blood glucose monitoring isn't indicated in this situation because the client doesn't have diabetes mellitus. Hydrocortisone replacement doesn't cause insulin resistance.

A client with type 1 diabetes presents with a decreased level of consciousness and a fingerstick glucose level of 39 mg/dl. His family reports that he has been skipping meals in an effort to lose weight. Which nursing intervention is most appropriate?

Administering 1 ampule of 50% dextrose solution, per physician's order

A client is experiencing an acute exacerbation of rheumatoid arthritis. What should the nursing priority be?

Administering ordered analgesics and monitoring their effects

A client with a history of hypertension is diagnosed with primary hyperaldosteronism. This diagnosis indicates that the client's hypertension is caused by excessive hormone secretion from which gland?

Adrenal cortex Excessive secretion of aldosterone in the adrenal cortex is responsible for the client's hypertension. This hormone acts on the renal tubule, where it promotes reabsorption of sodium and excretion of potassium and hydrogen ions. The pancreas mainly secretes hormones involved in fuel metabolism. The adrenal medulla secretes the catecholamines — epinephrine and norepinephrine. The parathyroids secrete parathyroid hormone.

The nurse is caring for a client with rheumatoid arthritis who suffers with chronic pain in the hands. When would be the best time for the nurse to perform range-of-motion exercises?

After the client has had a warm paraffin hand bath

A nurse is caring for a diabetic patient with a diagnosis of nephropathy. What would the nurse expect the urinalysis report to indicate?

Albumin

The side effect of bone marrow depression may occur with which medication used to treat gout?

Allopurinol

Which term refers to fixation or immobility of a joint?

Ankylosis

When assisting the patient to interpret a negative HIV test result, the nurse informs the patient that the results mean which of the following?

Antibodies to HIV are not present in his blood.

A client comes into the emergency department reporting difficulty walking and loss of muscle control in the arms. Once the nurse begins the physical examination, which assessment should be completed if an immune dysfunction in the neurosensory system is suspected?

Assess for ataxia using the finger-to-nose test and heel-to-shin test

A client is admitted to the health care center with abdominal pain, nausea, and vomiting. The medical reports indicate a history of type 1 diabetes. The nurse suspects the client's symptoms to be those of diabetic ketoacidosis (DKA). Which action will help the nurse confirm the diagnosis?

Assess the client's breath odor

During the first 24 hours after a client is diagnosed with addisonian crisis, which intervention should the nurse perform frequently?

Assess vital signs. Because the client in addisonian crisis is unstable, vital signs and fluid and electrolyte balance should be assessed every 30 minutes until he's stable. Daily weights are sufficient when assessing the client's condition. The client shouldn't have ketones in his urine, so there is no need to assess the urine for their presence. Oral hydrocortisone isn't administered during the first 24 hours in severe adrenal insufficiency.

The nurse identifies a nursing diagnosis of ineffective airway clearance related to pneumocystis pneumonia and increased bronchial secretions for a client with AIDS. Which of the following would be appropriate for the nurse to include in the client's plan of care?

Assist with chest physiotherapy every 2 to 4 hours.

A nurse is preparing an in-service presentation about human immunodeficiency virus (HIV) for a group of new graduate nurses. As part of the presentation, the nurse is planning to describe the events that occur once HIV enters the host cell. Which of the following would the nurse describe as the first step?

Attachment

A client who has AIDS reports having diarrhea after every meal, and wants to know what can be done to stop this symptom. What should the nurse advise?

Avoid fibrous foods, lactose, fat, and caffeine.

Which of the following would the nurse prioritize as the most important action for the patient to take to prevent anaphylaxis?

Avoid potential allergens

A nurse is assessing a client who has DM and reports feeling anxious. Which of the following findings should the nurse expect if the client is hypoglycemic? A. Rapid, deep respirations B. Cool, clammy skin C. Abdominal cramping D. Orthostatic Hypotension

B.

A nurse is assessing a client who has diabetes insipidus. The nurse should expect which of the following findings? A. Decreased heart rate B. Increased Hematocrit C. High urine specific gravity D. Decreased BUN

B.

A nurse is caring for a client who has diabetes mellitus and developed peripheral neuropathy. Which of the following measures should the nurse recommend to prevent injuries to his feet? A. Examine the skin and feet weekly for alterations in skin integrity. B. Monitor the temperature of bath water with a thermometer. C. Shop for shoes early in the day D. Round the edges of toenails when trimming.

B.

A nurse is planning dietary teaching for a client who has type 1 diabetes mellitus. Which of the following information should the nurse include regarding alcohol consumption? A, Substitute two carbohydrate exchanges for every one alcoholic bev. B. Ingest alcohol with meals to reduce alcohol-induced hypoglycemia. C. Consuming alcohol decreases blood triglyceride levels. D. Expect to increase insulin dosage when consuming alcohol.

B.

A nurse is providing discharge teaching for a client who has DI and has a new scrip for desmopressin nasal spray. Which of the following instructions should the nurse include in the teaching? A. Breath deeply while using the nasal spray B. Blow nose gently prior to using the nasal spray. C. Administer the spray while in a side-lying position. D. Instill the med 4x per day.

B.

A nurse is providing teachings for a client who is scheduled for a Pap test. The nurse should instruct the client that she is being tested for which of the following? A. Uterine cancer B. Cervical cancer C. Ovarian cyst D. Fibroids

B.

A nurse is teaching a client about glycosylated hemoglobin (HbA1c) testing. Which of the following statements by the client indicates an understanding of the information about this test? A. "I need to fast after midnight the night before the test." B. "This test is a good indicator of my average blood glucose levels." C. "A level of 8% to 10% suggests adequate blood glucose control." D. "I will use my hemoglobin A1c level to adjust my daily insulin doses."

B.

A nurse is caring for a client who has hodgkin's lymphoma. Which of the following fidnings should the nurse expect? A. Overgrowth of B-lymphocyte plasma cells B. Reed-Sternberg cells C. Epstein-Barr virus D. Overproduction of blast phase cells

B. are cancer cells specific to Hodgkin's lymphoma and are found in lymph nodes

A nurse is providing discharge teaching for a client who is HIV-positive. Which of the following instructions should the nurse include int he teaching? A. Clean bathroom surfaces with full-strength bleach B. Discard beverages that have been unrefrigerated for 1 hour C. Wash laundry soiled with a body fluid in warm water. D. Work in the garden for exercise

B. b/c they can support bacteria

A nurse is performing a breast examination on a client. Which of the following should the nurse report to the provider? A. Asymmetrical breast size B. Breast tissue with an orange-peel appearance C. Presence of Montgomery's tubercles on the aureola D. Moveable mass in the left-lower breast quadrant

B. d/t lymph channels indicates advanced breast cancer

A nurse if caring for a client who has viral pneumonia and a history of COPD. Which of the following finding should the nurse report to the provider? A. Consolidation in lower lobes by chest x ray B. Left shift in the WBC differential C. Oxygen sat 91% D. Orthostatic hypotension

B. indicated that the pneumonia is of bacterial origin, rather than viral

A nurse in the emergency department is assessing a newly admitted client. Which of the following places the client at increased risk for contracting hepatitis B? A. Residing in an institutional setting B. Engaging in unprotected sexual intercourse C. Working w/ hazardous chemical waste materials D. Traveling to a foreign country

B. transmitted by sexual contact

A child is brought into the emergency department with vomiting, drowsiness, and blowing respirations. The father reports that the symptoms have been progressing throughout the day. The nurse suspects diabetic ketoacidosis (DKA). Which action should the nurse take first in the management of DKA?

Begin fluid replacements Management of DKA is aimed at correcting dehydration, electrolyte loss, and acidosis before correcting the hypoglycemia with insulin.

A client is admitted with hyperosmolar hyperglycemic nonketotic syndrome (HHNS). Which laboratory finding should the nurse expect in this client?

Blood glucose level 1,100 mg/dl The blood glucose level rises to above 600 mg/dl in response to illness or infection

A patient is suspected of having a pheochromocytoma and is having diagnostic tests done in the hospital. What symptoms does the nurse recognize as most significant for a patient with this disorder?

Blood pressure varying between 120/86 and 240/130 mm Hg Hypertension associated with pheochromocytoma may be intermittent or persistent. Blood pressures exceeding 250/150 mm Hg have been recorded. Such blood pressure elevations are life threatening and can cause severe complications, such as cardiac dysrhythmias, dissecting aneurysm, stroke, and acute kidney failure.

Which condition is associated with impaired immunity relating to the aging client?

Breakdown and thinning of the skin

A nurse is admitting a client who has hyperthyroidism. When assessing the client, the nurse should expect which of the following findings? A. Cold intolerance B. Lethargy C. Tremors D. Sunken eyes

C.

A nurse is assessing a client who has a new diagnosis of Cushing's disease. Which of the following findings should the nurse expect? A. Decreased blood pressure B. Weight loss C. Hirsutism D. Increased skin thickness

C.

A nurse is assessing a client who has adrenal insufficiency. Which of the following findings should the nurse expect? A. Moon face B. Weight gain C. Serum calcium 12.8 mg/dL D. Serum sodium 150 mEq/L

C.

A nurse is caring for a client following a thyroidectomy. The nurse should assess for which of the following findings as an indication of hypocalcemia? A. Strong, bounding pulse B. Decreased bowel sounds. C. Tingling and numbness of the hands and feet. D. Diminished deep tendon reflexes.

C.

A nurse is caring for a client who has HIV. Which of the following laboratory findings should suggest to the nurse that medication therapy is effective? A. WBC 3,500 B. Lymphocyte 1,500 C. Decreased viral load D. Low CD4/CD8 ratio

C.

A nurse is providing teaching for a client who has type 1 DM about to prevent complications during illness. Which of the following statements indicates that the client understands the teaching? A. I should stop taking insulin if i feel nauseous B. I will test my urine for protein when I start to feel ill. C. I will call my doctor if my blood sugar is more than 250 mg/dL D. I should check my blood glucose level every 8 hours.

C.

A nurse is educating a client who is scheduled for a kidney transplant. Which of the following information regarding hyper-acute rejection should the nurse include in the teaching? A. Hyperacute rejection can occur during the first few weeks after the transplant. B. If hyperacute rejection occurs, the kidney can become enlarged. C. The organ will need to be removed if hyeracute rejection occurs. D. Immunosupressive therapy is given to reverse hyperacute rejection.

C. It's the only treatment

A nurse is planning an education program about testicular cancer for a group of male adolescents. Which of the following information should the nurse include? A. Testicular cancer is more common in men older than 65. B. With early treatment, the survival is 50% C. Examine your testicles immediately after showering. D. Schedule a yearly ultrasound to screen for testicular cancer.

C. b/c it's easier to palpate

A nurse is caring for a pt who has an elevated prostate-specific antigen level. The nurse should anticipate that the client will undergo which of the following diagnostic tests? A. Palpation of testes and lymph nodes B. Human chorionic gonadotropin level C. Digital rectal examination D. Pelvic ultrasound

C. determines size and consistency of the prostate

A nurse is providing teaching for a client who has an allergy to peanuts. Which if the following instructions is the priority to include in the teaching? A. Inform other health care professionals of the allergy B. Wear a medical identification tag C. Carry an emergency anaphylaxis kit D. Read food labels

C. greatest risk to client is injury from an anaphylactic reaction

A nurse is providing teaching for a client who has systemic lupus erythematosus (SLE). Which of the following statements by the client indicates an understanding of the teaching? A. "I should use a suncreen with an SPF of at least 15" B. "Long-term immunosuppresive therapy could cure this disease" C. " I should wear gloves when it is cold outside" D. "SLE should not affect my lungs or breathing"

C. raynaud's syndrome commonly accompanies SLE and can cause painful vasoconstriction in the fingers when they are exposed to cols temps

A nurse is caring for a client who is receiving chemotherapy and has laboratory data revealing bone marrow suppression. The nurse should include which of the following instructions in the teaching? A. take aspirin for minor aches and pains B. Rinse the toothbrush with warm water and after each use C. Avoid eating fresh fruit and vegetables D. Wear clothing that will minimize sun exposure

C. they can contain bacteria

The nurse is educating a patient about the risks of stroke related to the new prescription for a COX-2 inhibitor and what symptoms they should report. Which COX-2 inhibitor is the nurse educating the patient about?

Celecoxib (Celebrex)

A client with type 1 diabetes reports waking up in the middle of the night feeling nervous and confused, with tremors, sweating, and a feeling of hunger. Morning fasting blood glucose readings have been 110 to 140 mg/dL. The client admits to exercising excessively and skipping meals over the past several weeks. Based on these symptoms, the nurse plans to instruct the client to

Check blood glucose at 3:00 a.m. In the Somogyi effect, the client has a normal or elevated blood glucose concentration at bedtime, which decreases to hypoglycemic levels at 2 to 3 a.m., and subsequently increases as a result of the production of counter-regulatory hormones. It is important to check blood glucose in the early morning hours to detect the initial hypoglycemia.

The nurse practitioner who is monitoring the patient's progression of HIV is aware that the most debilitating gastrointestinal condition found in up to 90% of all AIDS patients is:

Chronic diarrhea.

The nurse intervenes to assist the client with fibromyalgia to cope with which symptoms?

Chronic fatigue, generalized muscle aching, and stiffness

Which factors will cause hypoglycemia in a client with diabetes? Select all that apply.

Client has not consumed food and continues to take insulin or oral antidiabetic medications. Client has not consumed sufficient calories. Client has been exercising more than usual.

The nurse is evaluating the plan of care for a client with an allergic disorder who has a nursing diagnosis of deficient knowledge related to measures for allergy control. Which of the following would indicate to the nurse that the outcome has been met?

Client identifies methods for reducing exposure risk to allergens.

A client who was diagnosed with scleroderma 2 years ago is withdrawn, does not look at the nurse, and asks to be left alone. An appropriate nursing diagnosis for the client is:

Disturbed body image

The treatment of gout involves managing the acute inflammatory stage, preventing flare-ups, and controlling hyperuricemia. Select the agent of first choice when an acute inflammatory attack begins.

Colchicine

When caring for a client experiencing an acute gout attack, the nurse anticipates administering which medication?

Colchicine

A client's blood glucose level is 45 mg/dl. The nurse should be alert for which signs and symptoms?

Coma, anxiety, confusion, headache, and cool, moist skin Signs and symptoms of hypoglycemia (indicated by a blood glucose level of 45 mf/dl) include anxiety, restlessness, headache, irritability, confusion, diaphoresis, cool skin, tremors, coma, and seizures.

A client is admitted to the hospital with a diagnosis of pneumonia. The client informs the nurse that he has several drug allergies. The physician has ordered an antibiotic as well as several other medications for cough and fever. What should the nurse do prior to administering the medications?

Consult drug references to make sure the medicines do not contain substances which the client is hypersensitive.

What interventions can the nurse encourage the client with diabetes insipidus to do in order to control thirst and compensate for urine loss?

Consume adequate amounts of fluid. The nurse teaches the client to consume sufficient fluid to control thirst and to compensate for urine loss. The client will not be required to come in daily for IV fluid therapy. The client should not limit fluid intake at night if thirst is present. Weighing daily will not control thirst or compensate for urine loss.

A nurse educates a group of clients with diabetes mellitus on the prevention of diabetic nephropathy. Which of the following suggestions would be most important?

Control blood glucose levels.

An experiment is designed to determine specific cell types involved in cell-mediated immune response. The experimenter is interested in finding cells that attack the antigen directly by altering the cell membrane and causing cell lysis. Which cells should be isolated?

Cytotoxic T cells

The actions of parathyroid hormone (PTH) are increased in the presence of which vitamin?

D The actions of PTH are increased by the presence of vitamin D.

A nurse is providing care to a female client with atopic dermatitis who has numerous lesions on the arms, neck, and legs. The client states, "My skin is so ugly. I can't imagine what people must think when they look at me. I can't look at myself sometimes." When developing the plan of care for this client, which nursing diagnosis would be most appropriate?

Disturbed body image related to appearance of visible lesions

A nurse is caring for a client undergoing evaluation for possible immune system disorders. Which intervention will best help support the client throughout the diagnostic process?

Educate the client about the diagnostic procedures and answer their questions about the possible diagnosis

A HH nurse is assessing a client who is on lifelong hormone replacement therapy for tx of hypothyroidism. Client has not been taking meds regularly, Which of the following findings should the nurse expect? A. Increased urine output B. Persistant diarrhea C. Tachycardia D. Hypotension

D.

A nurse in an outpatient clinic is teaching a client who has a diabetic foot ulcer about foot care. Which of the following statements by the client indicates understanding? A. I will let my feet air dry after washing B. I will wear sandals to allow air to circulate around my feet. C. I will buy OTC medicine to tx the calluses on my feet. D. I will apply lotion to the dry areas of my feet, avoiding application between my toes.

D.

A nurse is caring for a client undergoing screening for primary Cushing's disease. The nurse should expect that which of the following laboratory findings to be elevated? A. Lymphocyte B. Serum Potassium C. Serum Calcium D. Blood Glucose

D.

A nurse is caring for a client who has type 2 diabetes melliltus and has hyperglycemic-hyperosmolar state (HHS). Which of the following laboratory findings should the nurse expect? A. Serum pH of 7.32 B. Blood glucose of 250 mg/dL C. Blood glucose of 425 mg/dL D. Serum pH of 7.45

D.

A nurse is preparing a teaching plan for a client who has DI and is receiving intranasal desmopressin. Which of the following should the nurse include in the teaching plan? A. Drink at least 3L of fluid per day B. Weigh yourself weekly while wearing similar clothing at the same time of day C. Notify the provider of a weight loss of 0.45 kg (1lb) or more per week D. Report nocturia bc it requires a dosage adjustment

D.

A nurse is preping to give a client information about an ACTH stimulation test. The nurse should explain that the purpose of the test is to assess for which of the following disorders? A. Cushing's syndrome B. Hyperthyroidism C. Pheochromocytoma D. Addison's disease

D.

A nurse is providing care for four clients. Which of the following clients is at the greatest risk for pneumonia? A. A school-age child who has a history of allergies and asthma B. A young adult client living in a college dormitory C. A middle-age adult using an incentive spirometer following surgery D. An older adult client transferred from a long-term care facility who has dysphagia

D.

A nurse is providing teaching for a client who has diabetes mellitus. Which of the following findings associated with diabetic ketoacidosis (DKA) should the nurse include? A. Decreased urine output B. Weight gain of 0.45 kg (1 lb) in 24 hr C. Rapid, shallow respirations D. Blood glucose levels greater than 300 mg/dL

D.

To screen a client for pheochromocytoma, a nurse schedules a vanillylmandelic acid test. When teaching the client about this test, which of the following instructions should the nurse include? A. Start fasting at midnight prior to the day of the test. B. Begin the 24-hr urine collection with the first morning urination. C. Take low-dose aspirin for pain during the testing period. D. Restrict coffee intake 2 to 3 days prior to the test.

D.

A nurse is providing teaching for a client who has rheumatoid arthritis and reports persistent pain. Which of the following responses by the nurse is appropriate? A. "Take a cool bath in the evening." B. "Exercise every other day" C. "Use pillows to support your joints while in bed" D. "Ask family members to help with household chores"

D. Gives the client an opportunity for rest

A nurse is caring for a client who has non-Hodgkin's lymphoma and is receiving chemotherapy. Which of the following is the priority assessment finding? A. Loss of body hair B. Report of anorexia C. Mucositis of the oral cavity D. Erythema at the IV insertion site

D. Greatest risk is injury to the tissue due to extravasation and infection

Which information should be included in the teaching plan for a client receiving glargine, which is "peakless" basal insulin?

Do not mix with other insulins. Because glargine is in a suspension with a pH of 4, it cannot be mixed with other insulins because this would cause precipitation. Glargine is a "peakless" basal insulin that is absorbed very slowly over a 24-hour period and can be given once a day.

A nurse is planning an education program for a grp of high school teachers who will be taking students on a hike. Which of the following info should the nurse unclouded regarding Lyme disease? A. "If bitten by a tick, you should get tested immediately" B. "If you have a tick embedded in your skin, apply a lit match to remove it" C. "You should wear dark-colored clothing to deter ticks from biting" D. "If you develop pain and stiffness in your joints, you should see your doctor"

D. Stage 1 lyme disease, "bull's eye" rash, muscle and joint pain and stiffness

A nurse is caring for a client who has leukemia and a platelet count of 48.000. Which of the following actions should the nurse take? A. Provide a diet low in vit K B. Place the client on contact precautions C. Admin SQ epoetin alfa D. test urine and stool for occult blood

D. Thrombocytopenic is at risk for occult bleeding

A nurse is providing teaching for a client who has Hodgkin's lymphoma and is undergoing external radiation treatment. Which of the following instructions should the nurse include? A. Use an antibacterial soap to cleanse the skin. B. wash the ink marking off when showering C. Rub the skin with a towel when drying D. Avoid direct sunlight exposure to the skin

D. it can be damaging to skin being exposed to additional radiation

A nurse is providing education for the parent of a child about administration guidelines for the human papilloma virus (HPV) vaccine. Which of the following info should the nurse include? A. One dose should be given at birth and another at 5 years B. The vaccine does not protect males C. The vaccine protects against chlamydia D. Three doses should be given starting at age 11 or 12

D. second given 1 to 2 months after 1st dose and third given 6 months after 1st dose

A nurse is planning care for a client in acute addisonian crisis. Which nursing diagnosis should receive the highest priority?

Decreased cardiac output An acute addisonian crisis is a life-threatening event, caused by deficiencies of cortisol and aldosterone. Glucocorticoid insufficiency causes a decrease in cardiac output and vascular tone, leading to hypovolemia. The client becomes tachycardic and hypotensive and may develop shock and circulatory collapse. The client with Addison's disease is at risk for infection; however, reducing infection isn't a priority during an addisonian crisis. Impaired physical mobility and Imbalanced nutrition: Less than body requirements are appropriate nursing diagnoses for the client with Addison's disease, but they aren't priorities in a crisis.

During assessment of a patient with Kaposi's sarcoma, the nurse knows to look for the initial sign of:

Deep purple cutaneous lesions.

A client with long-standing type 1 diabetes is admitted to the hospital with unstable angina pectoris. After the client's condition stabilizes, the nurse evaluates the diabetes management regimen. The nurse learns that the client sees the physician every 4 weeks, injects insulin after breakfast and dinner, and measures blood glucose before breakfast and at bedtime. Consequently, the nurse should formulate a nursing diagnosis of:

Deficient knowledge (treatment regimen). The client should inject insulin before, not after, breakfast and dinner — 30 minutes before breakfast for the a.m. dose and 30 minutes before dinner for the p.m. dose.

A client received 2 units of packed red blood cells while in the hospital with rectal bleeding. Three days after discharge, the client experienced an allergic response and began to itch and break out with hives. What type of reaction does the nurse understand could be occurring?

Delayed hypersensitivity response

A client with an allergic disorder calls the nurse and asks what treatment is available for allergic disorders. The nurse explains to the client that there is more than one treatment available. What treatments would the nurse tell the client about?

Desensitization

A nurse is preparing a client with type 1 diabetes for discharge. The client can care for himself; however, he's had a problem with unstable blood glucose levels in the past. Based on the client's history, he should be referred to which health care worker?

Dietitian

A patient is experiencing an allergic reaction to a dose of penicillin. What should the nurse look for in the patient's initial assessment?

Dyspnea, bronchospasm, and/or laryngeal edema.

A school nurse is talking about infection with a high school health class. What would be the nurse's best explanation of the process of phagocytosis?

Engulfment and digestion of bacteria and foreign material

Which blood test confirms the presence of antibodies to HIV?

Enzyme-linked immunosorbent assay (ELISA)

A client with chronic renal failure has begun treatment with a colony-stimulating factor. What medication does the nurse anticipate administering to the client that will promote the production of blood cells?

Epoetin alfa (Epogen)

A nurse assesses a client in the physician's office. Which assessment findings support a suspicion of systemic lupus erythematosus (SLE)?

Facial erythema, pericarditis, pleuritis, fever, and weight loss

The client with rheumatoid arthritis has a red blood cell count of 3.2 cells/cu mm. Which nursing diagnosis has the highest priority for the client?

Fatigue

A health care provider prescribed Flonase, an intranasal corticosteroid, for a patient with a severe case of allergic rhinitis. The nurse told the patient that there is a delayed response to full benefits from the drug. The patient took his first dose on February 2. The patient should expect that the drug will be fully effective no later than:

February 15.

A health care provider suspects that a thyroid nodule may be malignant. The nurse knows to prepare information for the patient based on the usual test that will be ordered to establish a diagnosis. What is that test?

Fine-needle biopsy of the thyroid gland Fine needle biopsy of the thyroid gland is often used to establish the diagnosis of thyroid cancer. The purpose of the biopsy is to differentiate cancerous thyroid nodules from noncancerous nodules and to stage the cancer if detected. The procedure is safe and usually requires only a local anesthetic.

Nursing students are reviewing various medications that can be used to treat allergic disorders. The students demonstrate understanding of the information when they identify which of the following as an intranasal corticosteroid?

Fluticasone

The nurse is preparing a teaching plan for a client with an immunodeficiency. Which of the following would the nurse emphasize as most important?

Frequent and thorough handwashing

Which assessment finding is most important in determining nursing care for a client with diabetes mellitus?

Fruity breath

A 38-year-old client has been diagnosed with rheumatoid arthritis, an autoimmune disease. During the health history assessment. the nurse learns that the client works as an aide at a facility that cares for children infected with AIDS, does moderate cardiovascular exercises every other day, takes no medication, has no allergies, and eats mainly a vegetarian diet with fish and chicken one to two times each week. Which factor is the most important consideration in determining the status of the client's immune system?

Gender

The nurse is reviewing a client's history which reveals that the client has had an over secretion of growth hormone (GH) that occurred before puberty. The nurse interprets this as which of the following?

Gigantism When over secretion of GH occurs before puberty, gigantism results. Dwarfism occurs when secretion of GH is insufficient during childhood. Oversecretion of GH during adulthood results in acromegaly. An absence of pituitary hormonal activity causes Simmonds' disease.

The nurse is reviewing a client's history which reveals that the client has had an oversecretion of growth hormone (GH) that occurred before puberty. The nurse interprets this as which of the following?

Gigantism When oversecretion of GH occurs before puberty, gigantism results. Dwarfism occurs when secretion of GH is insufficient during childhood. Oversecretion of GH during adulthood results in acromegaly. An absence of pituitary hormonal activity causes Simmonds' disease.

Which of the following insulins are used for basal dosage?

Glarginet (Lantus) Lantus is used for basal dosage. NPH is an intermediate acting insulin, usually taken after food. Humalog and Novolog are rapid-acting insulins.

A nurse is teaching a client with type 1 diabetes how to treat adverse reactions to insulin. To reverse hypoglycemia, the client ideally should ingest an oral carbohydrate. However, this treatment isn't always possible or safe. Therefore, the nurse should advise the client to keep which alternate treatment on hand?

Glucagon

During a follow-up visit 3 months after a new diagnosis of type 2 diabetes, a client reports exercising and following a reduced-calorie diet. Assessment reveals that the client has only lost 1 pound and did not bring the glucose-monitoring record. Which value should the nurse measure?

Glycosylated hemoglobin level

A woman with a progressively enlarging neck comes into the clinic. She mentions that she has been in a foreign country for the previous 3 months and that she didn't eat much while she was there because she didn't like the food. She also mentions that she becomes dizzy when lifting her arms to do normal household chores or when dressing. What endocrine disorder should the nurse expect the physician to diagnose?

Goiter A goiter can result from inadequate dietary intake of iodine associated with changes in foods or malnutrition. It's caused by insufficient thyroid gland production and depletion of glandular iodine. Signs and symptoms of this malfunction include an enlarged thyroid gland, dizziness when raising the arms above the head, dysphagia, and respiratory distress. Signs and symptoms of diabetes mellitus include polydipsia, polyuria, and polyphagia. Signs and symptoms of diabetes insipidus include extreme polyuria (4 to 16 L/day) and symptoms of dehydration (poor tissue turgor, dry mucous membranes, constipation, dizziness, and hypotension). Cushing's syndrome causes buffalo hump, moon face, irritability, emotional lability, and pathologic fractures.

A middle-aged female client complains of anxiety, insomnia, weight loss, the inability to concentrate, and eyes feeling "gritty." Thyroid function tests reveal the following: thyroid-stimulating hormone (TSH) 0.02 U/ml, thyroxine 20 g/dl, and triiodothyronine 253 ng/dl. A 6-hour radioactive iodine uptake test showed a diffuse uptake of 85%. Based on these assessment findings, the nurse should suspect:

Graves' disease. Graves' disease, an autoimmune disease causing hyperthyroidism, is most prevalent in middle-aged females. In Hashimoto's thyroiditis, the most common form of hypothyroidism, TSH levels would be high and thyroid hormone levels low. In thyroiditis, radioactive iodine uptake is low (?2%), and a client with a multinodular goiter will show an uptake in the high-normal range (3% to 10%).

Which of the following indicates that a client with HIV has developed AIDS?

Herpes simplex ulcer persisting for 2 months

Which is the best nursing explanation for the symptom of polyuria in a client with diabetes mellitus?

High sugar pulls fluid into the bloodstream, which results in more urine production.

Which body substance causes increased gastric secretion, dilation of capillaries, and constriction of the bronchial smooth muscle?

Histamine

Which is a primary chemical mediator of hypersensitivity?

Histamine

A nurse practitioner working in a dermatology clinic explained to a group of nursing students that the pathophysiology of an allergic response involves a chain of events that includes responses from lymphocytes, IgE, mast cells, and basophils. The nurse mentioned that the most important chemical mediator involved in the response is:

Histamine.

Which condition should a nurse expect to find in a client diagnosed with hyperparathyroidism?

Hypercalcemia Hypercalcemia is the hallmark of excess parathyroid hormone levels. Serum phosphate will be low (hypophosphatemia), and there will be increased urinary phosphate (hyperphosphaturia) because phosphate excretion is increased.

Which of the following suggests to the nurse that the client with systemic lupus erythematous is having renal involvement?

Hypertension

The nurse is assessing a client in the clinic who appears restless, excitable, and agitated. The nurse observes that the client has exophthalmos and neck swelling. What diagnosis do these clinical manifestations correlate with?

Hyperthyroidism Clients with hyperthyroidism characteristically are restless despite felling fatigued and weak, highly excitable, and constantly agitated. Fine tremors of the hand occur, causing unusual clumsiness. The client cannot tolerate heat and has an increased appetite but loses weight. Diarrhea also occurs. Visual changes, such as blurred or double vision, can develop. Exophthalmos, seen in clients with severe hyperthyroidism, results from enlarged muscle and fatty tissue surrounding the rear and sides of the eyeball. Neck swelling caused by the enlarged thyroid gland often is visible. Hypothyroidism clinical manifestations are the opposite of what is seen as hyperthyroidism. SIADH and DI clinical manifestations do not correlate with the symptoms manifested by the client.

During a third-trimester transabdominal ultrasound, cardiac anomaly and facial abnormalities are noted in the fetus. Further testing reveals that the thymus gland has failed to develop normally, and the fetus is diagnosed with thymic hypoplasia. Based on this diagnosis, the nurse anticipates careful monitoring for which common manifestation during the first 24 hours of life?

Hypocalcemia

A client who is HIV positive is experiencing severe diarrhea. Which laboratory test result would the nurse expect to find?

Hypokalemia

Which combination of adverse effects should a nurse monitor for when administering IV insulin to a client with diabetic ketoacidosis?

Hypokalemia and hypoglycemia

A client with acromegaly has been given the option of a surgical approach or a medical approach. The client decides to have a surgical procedure to remove the pituitary gland. What does the nurse understand this surgical procedure is called?

Hypophysectomy The treatment of choice is surgical removal of the pituitary gland (transsphenoidal hypophysectomy) through a nasal approach. The surgeon may substitute an endoscopic technique using microsurgical instruments to reduce surgical trauma. A hysteroscopy is a gynecologic procedure. The thyroid gland is not involved for a surgical procedure. Ablation is not a removal of the pituitary gland.

A client has been diagnosed with myxedema from long-standing hypothyroidism. What clinical manifestations of this disorder does the nurse recognize are progressing to myxedema coma? Select all that apply.

Hypothermia Hypotension Hypoventilation Severe hypothyroidism is called myxedema. Advanced, untreated myxedema can progress to myxedemic coma. Signs of this life-threatening event are hypothermia, hypotension, and hypoventilation. Hypertension and hyperventilation indicate increased metabolic responses, which are the opposite of what the client would be experiencing.

A client with ataxia-telangiectasia is admitted to the unit. The nurse caring for the client would expect to see what included in the treatment regimen?

IV gamma globulin administration

A client is being discharged from the hospital after being diagnosed with and treated for systemic lupus erythematosus (SLE). What would the nurse not include when teaching the client and family information about managing the disease?

If you have problems with a medication, you may stop it until your next physician visit.

What type of immunoglobulin does the nurse recognize that promotes the release of vasoactive chemicals such as histamine when a client is having an allergic reaction?

IgE

There are several types of reactions to latex. The nurse knows to be most concerned about laryngeal edema with the following reaction:

IgE-mediated hypersensitivity.

A client with severe combined immunodeficiency is to receive a hematopoietic stem cell transplant. Which of the following would the nurse expect to be started?

Immunosuppressive agents

A nurse is assessing a client who is receiving total parenteral nutrition (TPN). Which finding suggests that the client has developed hyperglycemia?

Increased urine output

The client who is managing diabetes through diet and insulin control asks the nurse why exercise is important. Which is the best response by the nurse to support adding exercise to the daily routine?

Increases ability for glucose to get into the cell and lowers blood sugar

A nurse explains to a client that she will administer his first insulin dose in his abdomen. How does absorption at the abdominal site compare with absorption at other sites?

Insulin is absorbed more rapidly at abdominal injection sites than at other sites.

Which factor presents the most likely cause for weight gain in a diabetic client who is controlled with insulin?

Insulin is an anabolic hormone. Insulin is an anabolic hormone that is known to cause weight gain

The nurse is preparing a presentation for a group of adults at a local community center about diabetes. Which of the following would the nurse include as associated with type 2 diabetes?

Insulin production insufficient

NPH is an example of which type of insulin?

Intermediate-acting

The nurse working in an allergy clinic is preparing to administer skin testing to a client. Which route is the safest for the nurse to use to administer the solution?

Intradermal

Which of the following aid in diagnosing the risk of anaphylaxis?

Intradermal testing

The nurse assesses a patient who has an obvious goiter. What type of deficiency does the nurse recognize is most likely the cause of this?

Iodine Oversecretion of thyroid hormones is usually associated with an enlarged thyroid gland known as a goiter. Goiter also commonly occurs with iodine deficiency.

The nurse is describing the action of insulin in the body to a client newly diagnosed with type 1 diabetes. Which of the following would the nurse explain as being the primary action?

It enhances transport of glucose across the cell wall.

Which of the following statements is accurate regarding osteoarthritis?

It is a noninflammatory disorder and the most common and frequently disabling of joint disorders.

Which points should be included in the medication teaching plan for a client taking adalimumab?

It is important to monitor for injection site reactions.

Which findings best correlate with a diagnosis of osteoarthritis?

Joint stiffness that decreases with activity

Nursing students are reviewing information about the different manifestations associated with AIDS. The students demonstrate understanding of these manifestations when they identify which of the following as the most common HIV-related malignancy?

Kaposi's sarcoma

While caring for a patient with pneumocystis pneumonia, the nurse assesses flat, purplish lesions on the back and trunk. What does the nurse suspect these lesions indicate?

Kaposi's sarcoma

A patient is diagnosed with type 1 diabetes. What clinical characteristics does the nurse expect to see in this patient? Select all that apply.

Ketosis-prone Little endogenous insulin Younger than 30 years of age

Which of the following is considered the most serious manifestation of angioneurotic edema?

Laryngeal swelling

Which of the following would the nurse expect the physician to order for a client with hypothyroidism?

Levothyroxine sodium Hypothyroidism is treated with thyroid replacement therapy, in the form of desiccated thyroid extract or a synthetic product, such as levothyroxine sodium (Synthroid) or liothyronine sodium (Cytomel). Methimazole and propylthiouracil are antithyroid agents used to treat hyperthyroidism. Propranolol is a beta blocker that can be used to treat hyperthyroidism.

The nurse is educating the patient with gout about ways to prevent reoccurrence of an attack. What foods should the nurse encourage the patient to avoid?

Liver

A nursing student asks the instructor how to identify rheumatoid nodules in a client with rheumatoid arthritis. Which of the following characteristics would the instructor include?

Located over bony prominence

The nurse teaches the client that reducing the viral load will have what effect?

Longer survival

The nurse is caring for a client with an abnormally low blood glucose concentration. What glucose level will the nurse observe when assessing laboratory results?

Lower than 50 to 60 mg/dL (2.77 to 3.33 mmol/L)

A patient has enlarged lymph nodes in his neck and a sore throat. This inflammatory response is an example of a cellular immune response whereby:

Lymphocytes migrate to areas of the lymph node

The nursing students are learning about the immune system in their anatomy and physiology class. What would these students learn is a component of the immune system?

Lymphoid tissues

A T-cell deficiency is a disorder characterized by the following manifestations. Choose all that apply.

Lymphopenia is usually present. Chronic mucocutaneous candidiasis is an associated disorder.

The nurse is aware that the phagocytic immune response, one of the body's responses to invasion, involves the ability of cells to ingest foreign particles. Which of the following engulfs and destroys invading agents?

Macrophages

An obese Hispanic client, age 65, is diagnosed with type 2 diabetes. Which statement about diabetes mellitus is true?

More common in hispanics and blacks

A client with type 1 diabetes is experiencing polyphagia. The nurse knows to assess for which additional clinical manifestation(s) associated with this classic symptom?

Muscle wasting and tissue loss

A client with diabetes mellitus has a prescription for 5 units of U-100 regular insulin and 25 units of U-100 isophane insulin suspension (NPH) to be taken before breakfast. At about 4:30 p.m., the client experiences headache, sweating, tremor, pallor, and nervousness. What is the most probable cause of these signs and symptoms?

Serum glucose level of 52 mg/dl Headache, sweating, tremor, pallor, and nervousness typically result from hypoglycemia, an insulin reaction in which serum glucose level drops below 70 mg/dl

A client receiving thyroid replacement therapy develops influenza and forgets to take her thyroid replacement medicine. The nurse understands that skipping this medication puts the client at risk for developing which life-threatening complication?

Myxedema coma Myxedema coma, severe hypothyroidism, is a life-threatening condition that may develop if thyroid replacement medication isn't taken. Exophthalmos (protrusion of the eyeballs) is seen with hyperthyroidism. Although thyroid storm is life-threatening, it's caused by severe hyperthyroidism. Tibial myxedema (peripheral mucinous edema involving the lower leg) is associated with hypothyroidism but isn't life-threatening.

The nurse is preparing to administer intermediate-acting insulin to a patient with diabetes. Which insulin will the nurse administer?

NPH Intermediate-acting insulins are called NPH insulin (neutral protamine Hagedorn) or Lente insulin. Lispro (Humalog) is rapid acting, Iletin II is short acting, and glargine (Lantus) is very long acting.

A client with a tentative diagnosis of hyperosmolar hyperglycemic nonketotic syndrome (HHNS) has a history of type 2 diabetes that is being controlled with an oral diabetic agent, tolazamide. Which laboratory test is the most important for confirming this disorder?

Serum osmolarity Serum osmolarity is the most important test for confirming HHNS;

You are caring for a client who has a diagnosis of HIV. Part of this client's teaching plan is educating the client about his or her medications. What is essential for the nurse to include in the teaching of this client regarding medications?

Side effects of drug therapy

Which of the following glands is considered the master gland?

Pituitary Commonly referred to as the master gland, the pituitary gland secretes hormones that control the secretion of additional hormones by other endocrine glands. The thyroid, parathyroid, and adrenal glands are not considered the master gland.

The nurse's base knowledge of primary immunodeficiencies includes which of the following statements?

Primary immunodeficiencies develop early in life after protection from maternal antibodies decreases.

What is the function of the thymus gland?

Programs T lymphocytes to become regulator or effector T cells

A nurse is preparing the daily care plan for a client with newly diagnosed diabetes mellitus. The priority nursing concern for this client should be:

Providing client education at every opportunity

The nurse assesses a patient who has been diagnosed with Addison's disease. Which of the following is a diagnostic sign of this disease?

Potassium of 6.0 mEq/L Addison's disease is characterized by hypotension, low blood glucose, low serum sodium, and high serum potassium levels. The normal serum potassium level is 3.5 to 5 mEq/L.

A client is diagnosed with severe combined immunodficiency (SCID). Which of the following would the nurse expect to integrate into the client's plan of care?

Preparation for bone marrow transplantation

Which clinical characteristic is associated with type 1 diabetes (previously referred to as insulin-dependent diabetes mellitus)?

Presence of Islet cell antibodies Individuals with type 1 diabetes often have islet cell antibodies and are usually thin or demonstrate recent weight loss at the time of diagnosis. These individuals are prone to experiencing ketosis when insulin is absent and require exogenous insulin to preserve life.

A nurse is teaching a diabetic support group about the causes of type 1 diabetes. The teaching is determined to be effective when the group is able to attribute which factor as a cause of type 1 diabetes?

Presence of autoantibodies against islet cells

A client with acromegaly is admitted to the hospital with complaints of partial blindness that began suddenly. What does the nurse suspect is occurring with this client?

Pressure on the optic nerve Partial blindness may result from pressure on the optic nerve. Glaucoma does not occur suddenly, and the client did not report injury to suspect corneal abrasions or retinal detachment.

Which intervention is the single most important aspect for the client at risk for anaphylaxis?

Prevention

A client who was diagnosed with type 1 diabetes 14 years ago is admitted to the medical-surgical unit with abdominal pain. On admission, the client's blood glucose level is 470 mg/dl. Which finding is most likely to accompany this blood glucose level?

Rapid, thready pulse This client's abnormally high blood glucose level indicates hyperglycemia, which typically causes polyuria, polyphagia, and polydipsia. Because polyuria leads to fluid loss, the nurse should expect to assess signs of deficient fluid volume, such as a rapid, thready pulse; decreased blood pressure; and rapid respirations. Cool, moist skin and arm and leg trembling are associated with hypoglycemia. Rapid respirations — not slow, shallow ones — are associated with hyperglycemia.

The nurse is explaining glycosylated hemoglobin testing to a diabetic client. Which of the following provides the best reason for this order?

Reflects the amount of glucose stored in hemoglobin over past several months.

What is the only insulin that can be given intravenously?

Regular

A client is admitted to the unit with diabetic ketoacidosis (DKA). Which insulin would the nurse expect to administer intravenously?

Regular Regular insulin is administered intravenously to treat DKA.

The nurse is administering an insulin drip to a patient in ketoacidosis. What insulin does the nurse know is the only one that can be used intravenously?

Regular Short-acting insulins are called regular insulin (marked R on the bottle)

A patient with a history of anaphylactic reactions to insect stings has just been stung by a wasp. The patient is going to self-administer his EpiPen. Place the steps in the correct order that he would follow. Use all options.

Remove the gray safety-release cap Inject the black tip into the outer thigh Massage the injection area Call the emergency medical response number (911)

The client with rheumatoid arthritis is seen in the clinic. Which assessment finding indicates the client is having difficulty implementing self-care?

Reports increased fatigue

The nurse is reviewing the history and physical examination of a client diagnosed with hyperthyroidism. Which of the following would the nurse expect to find?

Reports of increased appetite Signs and symptoms of hyperthyroidism reflect the increased metabolic rate and would include reports of increased appetite, weight loss, and intolerance to heat. Sleepiness, thick hard nails, and intolerance to cold are associated with hypothyroidism.

A client with HIV will be started on a medication regimen of three medications. What drug will the nurse instruct the client about?

Reverse transcriptase inhibitors

A patient has been newly diagnosed with type 2 diabetes, and the nurse is assisting with the development of a meal plan. What step should be taken into consideration prior to making the meal plan?

Reviewing the patient's diet history to identify eating habits and lifestyle and cultural eating patterns

The nurse is completing the intake assessment of a client new to the allergy clinic. The client states that he was taking nose drops six times a day to relieve his nasal congestion. The nasal congestion increased, causing him to increase his usage of the nasal spray to eight times a day. But again the congestion worsened. The nurse communicates to the health care provider that the client experienced

Rhinitis medicamentosa

A nurse is poviding care for a client with progressive systemic sclerosis. For a client with this disease, the nurse is most likely to formulate which nursing diagnosis?

Risk for impaired skin integrity

A client has a decreased level of thyroid hormone being excreted. What will the feedback loop do to maintain the level of thyroid hormone required to maintain homeostatic stability?

Stimulate more hormones using the negative feedback system Feedback can be either negative or positive. Most hormones are secreted in response to negative feedback; a decrease in levels stimulates the releasing gland.

After teaching a group of students about systemic lupus erythematosus, the instructor determines that the teaching was successful when the students state which of the following?

The belief is that it is an autoimmune disorder with an unknown trigger.

A client on antiretroviral drug therapy is discussing with the nurse that sometimes he "forgets to take his meds for a few days." What should the nurse inform the client can occur when the medications are not taken as prescribed?

The client is risking the development of drug resistance and drug failure.

A postpartum client is receiving intravenous oxytocin (Pitocin) after birth. Why will this medication be used for this client after the birth of her child?

Stimulates the contraction of the uterus and prevents bleeding Oxytocin (Pitocin) is released from the pituitary gland and stimulates contraction of pregnant uterus and release of breast milk after childbirth. It will not prevent lactation or help treat nausea. It will increase lactation.

Thirty minutes after the nurse begins an intravenous immunoglobulin (IVIG) infusion, the client reports itching at the site and a lump in the throat. Which action should the nurse take first?

Stop the infusion.

In teaching clients with osteoarthritis about their condition, it would be important for the nurse to focus on:

Strategies for remaining active

A nurse is assigned to care for a postoperative client with diabetes mellitus. During the assessment interview, the client reports that he's impotent and says he's concerned about the effect on his marriage. In planning this client's care, the most appropriate intervention would be to:

Suggest referral to sex therapist/ other appropriate professional

A patient has a serum study that is positive for the rheumatoid factor. What does the nurse understand is the significance of this test result?

Suggestive of RA

A client with status asthmaticus requires endotracheal intubation and mechanical ventilation. Twenty-four hours after intubation, the client is started on the insulin infusion protocol. The nurse must monitor the client's blood glucose levels hourly and watch for which early signs and symptoms associated with hypoglycemia?

Sweating, tremors, and tachycardia

The nurse is working with a colleague who has a delayed hypersensitivity (type IV) allergic reaction to latex. Which statement describes the clinical manifestations of this reaction?

Symptoms are localized to the area of exposure, usually the back of the hands.

Which of the following disorders is characterized by an increased autoantibody production?

Systemic lupus erythematosus (SLE)

T-cell and B-cell lymphocytes are the primary participants in the immune response. What do they do?

T-cell and B-cell lymphocytes distinguish harmful substances and ignore those natural and unique to a person.

A male client, aged 42 years, is diagnosed with diabetes mellitus. He visits the gym regularly and is a vegetarian. Which of the following factors is important when assessing the client?

The client's consumption of carbohydrates While assessing a client, it is important to note the client's consumption of carbohydrates because he has high blood sugar.

A client with hypothyroidism (myxedema) is receiving levothyroxine (Synthroid), 25 mcg P.O. daily. Which finding should the nurse recognize as an adverse reaction to the drug?

Tachycardia Levothyroxine, a synthetic thyroid hormone, is given to a client with hypothyroidism to simulate the effects of thyroxine. Adverse reactions to this agent include tachycardia. Dysuria, leg cramps, and blurred vision aren't associated with levothyroxine.

The nurse is caring for a patient with hyperthyroidism who suddenly develops symptoms related to thyroid storm. What symptoms does the nurse recognize that are indicative of this emergency?

Temperature of 102ºF Thyroid storm is characterized by the following: 1) high fever (hyperpyrexia), >38.5°C (>101.3°F); 2) extreme tachycardia (>130 bpm); 3) exaggerated symptoms of hyperthyroidism with disturbances of a major system—for example, gastrointestinal (weight loss, diarrhea, abdominal pain) or cardiovascular (edema, chest pain, dyspnea, palpitations); and 4) altered neurologic or mental state, which frequently appears as delirium psychosis, somnolence, or coma.

Which instruction should a nurse give to a client with diabetes mellitus when teaching about "sick day rules"?

Test blood glucose every 4 hours The nurse should instruct a client with diabetes mellitus to check his blood glucose levels every 3 to 4 hours and take insulin or an oral antidiabetic agent as usual, even when he's sick. If the client's blood glucose level rises above 300 mg/dl, he should call his physician immediately. If the client is unable to follow the regular meal plan because of nausea, he should substitute soft foods, such as gelatin, soup, and custard.

A client has been diagnosed with prediabetes and discusses treatment strategies with the nurse. What can be the consequences of untreated prediabetes?

The NIDDK has developed criteria that identify people with prediabetes, which can lead to type 2 diabetes, heart disease, and stroke.

A client is scheduled for a diagnostic test to measure blood hormone levels. The nurse expects that this test will determine which of the following?

The functioning of endocrine glands Measuring blood hormone levels helps determine the functioning of endocrine glands. A radioimmunoassay determines the concentration of a substance in plasma. The measurement of blood hormone levels will not reveal a client's blood sugar level. Radiographs of the chest or abdomen determine the size of the organ and its location.

A patient taking corticosteroids for exacerbation of Crohn's disease comes to the clinic and informs the nurse that he wants to stop taking them because of the increase in acne and moon face. What can the nurse educate the patient regarding these symptoms?

The moon face and acne will resolve when the medication is tapered off. Cushing syndrome is commonly caused by the use of corticosteroid medications and is infrequently the result of excessive corticosteroid production secondary to hyperplasia of the adrenal cortex. The patient develops a "moon-faced" appearance and may experience increased oiliness of the skin and acne. If Cushing syndrome is a result of the administration of corticosteroids, an attempt is made to reduce or taper the medication to the minimum dosage needed to treat the underlying disease process (e.g., autoimmune or allergic disease, rejection of a transplanted organ).

The nurse is teaching a client who has been diagnosed with Hashimoto's thyroiditis. Which of the following statements correctly describes the process of autoimmunity?

The normal protective immune response attacks the body, damaging tissues.

A client is treated in the clinic for a sexually transmitted infection, and the nurse suspects that the client is at risk for HIV. The physician determines that the client should be tested for the virus. What responsibility does the nurse have?

The nurse ensures a written consent is obtained prior to testing.

The pancreas continues to release a small amount of basal insulin overnight, while a person is sleeping. The nurse knows that if the body needs more sugar:

The pancreatic hormone glucagon will stimulate the liver to release stored glucose. When sugar levels are low, glucagon promotes hyperglycemia by stimulating the release of stored glucose.

What education should the nurse provide to the patient taking long-term corticosteroids?

The patient should not stop taking the medication abruptly and should be weaned off of the medication.

A client has type 1 diabetes. Her husband finds her unconscious at home and administers glucagon, 0.5 mg subcutaneously. She awakens in 5 minutes. Why should her husband offer her a complex carbohydrate snack as soon as possible?

To restore liver glycogen and prevent secondary hypoglycemia

Which term indicates an accumulation of crystalline depositions in articular surfaces, bones, soft tissue, and cartilage?

Tophi

A client is returned to his room after a subtotal thyroidectomy. Which piece of equipment is most important for the nurse to keep at the client's bedside?

Tracheostomy set After a subtotal thyroidectomy, swelling of the surgical site (the tracheal area) may obstruct the airway. Therefore, the nurse should keep a tracheostomy set at the client's bedside in case of a respiratory emergency. Although an indwelling urinary catheter and a cardiac monitor may be used for a client after a thyroidectomy, the tracheostomy set is more important. A humidifier isn't indicated for this client.

A client with type 2 diabetes asks the nurse why he can't have a pancreatic transplant. Which of the following would the nurse include as a possible reason?

Underlying problem of insulin resistance

A nurse is preparing a discharge teaching plan for a client with atopic dermatitis. Which instruction should the nurse include in her teaching plan?

Use a topical skin moisturizer daily.

A 38-year-old client has begun to suffer from rheumatoid arthritis and is being assessed for disorders of the immune system. The client works as an aide at a facility that cares for children infected with AIDS. Which is the most important factor related to the client's assessment?

Use of other drugs

Which term defines the balance between the amount of HIV in the body and the immune response?

Viral set point

A nurse is assessing a client with hyperthyroidism. What findings should the nurse expect?

Weight loss, nervousness, and tachycardia Weight loss, nervousness, and tachycardia are signs of hyperthyroidism. Other signs of hyperthyroidism include exophthalmos, diaphoresis, fever, and diarrhea. Weight gain, constipation, lethargy, decreased sweating, and cold intolerance are signs of hypothyroidism.

A 16-year-old client newly diagnosed with type 1 diabetes has a very low body weight despite eating regular meals. The client is upset because friends frequently state, "You look anorexic." Which statement by the nurse would be the best response to help this client understand the cause of weight loss due to this condition?

Your body is using protein and fat for energy instead of glucose

For a client with hyperthyroidism, treatment is most likely to include:

a thyroid hormone antagonist. Thyroid hormone antagonists, which block thyroid hormone synthesis, combat increased production of thyroid hormone. Treatment of hyperthyroidism also may include radioiodine therapy, which destroys some thyroid gland cells, and surgery to remove part of the thyroid gland; both treatments decrease thyroid hormone production. Thyroid extract, synthetic thyroid hormone, and emollient lotions are used to treat hypothyroidism.

A client with diabetes mellitus develops sinusitis and otitis media accompanied by a temperature of 100.8° F (38.2° C). What effect do these findings have on his need for insulin?

They increase the need for insulin Insulin requirements increase in response to growth, pregnancy, increased food intake, stress, surgery, infection, illness, increased insulin antibodies, and some medications. Insulin requirements are decreased by hypothyroidism, decreased food intake, exercise, and some medications.

A client is prescribed an oral corticosteroid for 2 weeks to relieve asthma symptoms. The nurse educates the client about side effects, which include

adrenal suppression.

During a class on exercise for clients with diabetes mellitus, a client asks the nurse educator how often to exercise. To meet the goals of planned exercise, the nurse educator should advise the client to exercise:

at least three times per week.

A client has begun sensitivity testing to determine the allergen which caused an anaphylactic reaction 3 weeks ago. In scratch testing, which part of the body is more sensitive to allergens?

back

Kaposi sarcoma (KS) is diagnosed through

biopsy.

B lymphocytes mature in the __________; T lymphocytes mature in the _____________. Choose, in the correct order, the answer below.

bone marrow; thymus

Trousseau sign is elicited

by occluding the blood flow to the arm for 3 minutes with the use of a blood pressure cuff. A positive Trousseau sign is suggestive of latent tetany. A positive Chvostek sign is demonstrated when a sharp tapping over the facial nerve just in front of the parotid gland and anterior to the ear causes the mouth, nose, and eye to spasm or twitch. The palm remaining blanched when the radial artery is occluded demonstrates a positive Allen test. The radial artery should not be used for an arterial puncture. A positive Homans sign is demonstrated when the client reports pain in the calf when the foot is dorsiflexed.

A nurse visits the employee health department because of mild itching and a rash on both hands. During the assessment interview, the employee health nurse should focus on:

chemical and latex glove use.

The nurse is reviewing the medication administration record of the client. Which of the following medications would lead the nurse to suspect that the client is suffering from an acute attack of gout?

colchicine

A nurse is providing dietary instructions to a client with hypoglycemia. To control hypoglycemic episodes, the nurse should recommend:

consuming a low-carbohydrate, high-protein diet and avoiding fasting.

During a client education session, the nurse describes the mechanism of hormone level maintenance. What causes most hormones to be secreted?

decrease in hormonal levels Most hormones are secreted in response to negative feedback; a decrease in levels stimulates the releasing gland. In positive feedback, the opposite occurs.

A client is being evaluated for hypothyroidism. During assessment, the nurse should stay alert for:

decreased body temperature and cold intolerance. Hypothyroidism markedly decreases the metabolic rate, causing a reduced body temperature and cold intolerance. Other signs and symptoms include dyspnea, hypoventilation, bradycardia, hypotension, anorexia, constipation, decreased intellectual function, and depression. Exophthalmos; conjunctival redness; flushed, warm, moist skin; and a systolic murmur at the left sternal border are typical findings in a client with hyperthyroidism.

A nurse is assessing a client with possible Cushing's syndrome. In a client with Cushing's syndrome, the nurse expects to find:

deposits of adipose tissue in the trunk and dorsocervical area. Because of changes in fat distribution, adipose tissue accumulates in the trunk, face (moon face), and dorsocervical areas (buffalo hump). Hypertension is caused by fluid retention. Skin becomes thin and bruises easily because of a loss of collagen. Muscle wasting causes muscle atrophy and thin extremities.

A client with type 1 diabetes asks the nurse about taking an oral antidiabetic agent. The nurse explains that these medications are effective only if the client:

has type 2 diabetes.

A client has a known allergy to peanuts, meaning that the client's immune system has identified peanuts as a foreign invader and has produced specific cells to attack if the client should come in contact with peanuts again. The formation of these specific cells is known as:

humoral response

A client presents with a huge lower jaw, bulging forehead, large hands and feet, and frequent headaches. What could be causing this client's symptoms?

hyperpituitarism Acromegaly (hyperpituitarism) is a condition in which growth hormone is oversecreted after the epiphyses of the long bones have sealed. A client with acromegaly has coarse features, a huge lower jaw, thick lips, a thickened tongue, a bulging forehead, a bulbous nose, and large hands and feet. When the overgrowth is from a tumor, headaches caused by pressure on the sella turcica are common.

For the first 72 hours after thyroidectomy surgery, a nurse should assess a client for Chvostek's sign and Trousseau's sign because they indicate:

hypocalcemia. A client who has undergone a thyroidectomy is at risk for developing hypocalcemia from inadvertent removal of or damage to the parathyroid gland. The client with hypocalcemia will exhibit a positive Chvostek's sign (facial muscle contraction when the facial nerve in front of the ear is tapped) and a positive Trousseau's sign (carpal spasm when a blood pressure cuff is inflated for a few minutes). These signs aren't present with hypercalcemia, hypokalemia, or hyperkalemia.

A client with adrenal insufficiency is gravely ill and presents with nausea, vomiting, diarrhea, abdominal pain, profound weakness, and headache. The client's family reports that the client has been doing strenuous yard work all day and was sweating profusely. Nursing management of this client would include observation for signs of:

hyponatremia and hyperkalemia. The client's history and presenting symptoms suggest the onset of adrenal crisis. Laboratory findings that support adrenal deficiency and crisis include low serum sodium (hyponatremia) and high serum potassium (hyperkalemia) levels.

A nurse is providing education to a client who is newly diagnosed with diabetes mellitus. What are classic symptoms associated with diabetes?

increased thirst, hunger, and urination

A client has had a splenectomy after sustaining serious internal injuries in a motorcycle accident, including a ruptured spleen. Following removal of the spleen, the client will be susceptible to:

infection because the spleen removes bacteria from the blood.

What types of cells are the primary targets of the healthy immune system? Select all that apply.

infectious cells foreign cells cancerous cells

A client with a history of type 1 diabetes is demonstrating fast, deep, labored breathing and has fruity odored breath. What could be the cause of the client's current serious condition?

ketoacidosis

Although not designated as endocrine glands, several organs within the body secrete hormones as part of their normal function. Which organ secretes hormones involved in increasing blood pressure and volume and maturation of red blood cells?

kidneys The kidneys release renin, a hormone that initiates the production of angiotensin and aldosterone to increase blood pressure and blood volume. The kidneys also secrete erythropoietin, a substance that promotes the maturation of red blood cells.

An incoherent client with a history of hypothyroidism is brought to the emergency department by the rescue squad. Physical and laboratory findings reveal hypothermia, hypoventilation, respiratory acidosis, bradycardia, hypotension, and nonpitting edema of the face and periorbital area. Knowing that these findings suggest severe hypothyroidism, the nurse prepares to take emergency action to prevent the potential complication of:

myxedema coma. Severe hypothyroidism may result in myxedema coma, in which a drastic drop in the metabolic rate causes decreased vital signs, hypoventilation (possibly leading to respiratory acidosis), and nonpitting edema. Thyroid storm is an acute complication of hyperthyroidism. Cretinism is a form of hypothyroidism that occurs in infants. Hashimoto's thyroiditis is a common chronic inflammatory disease of the thyroid gland in which autoimmune factors play a prominent role.

Which immunity type becomes active as a result of infection by a specific microorganism?

naturally acquired active immunity

A nurse is caring for a client with a kidney disorder. What hormone released by the kidneys initiates the production of angiotensin and aldosterone to increase blood pressure and blood volume?

renin Renin is released from the kidneys and initiates the production of angiotensin and aldosterone to increase blood pressure and blood volume. The kidneys secrete erythropoietin, a substance that promotes the maturation of red blood cells. Cholecystokinin released from cells in the small intestine stimulates contraction of the gallbladder to release bile when dietary fat is ingested. Gastrin is released within the stomach to increase the production of hydrochloric acid.

A client has recently been diagnosed with rheumatoid arthritis, and is also receiving further testing for disorders of the immune system. The client works as an aide at a facility caring for children infected with AIDS. Which factors will hold significant implications during the client's assessment? Select all that apply.

work environment history of immunizations and allergies use of other drugs


Set pelajaran terkait

CHAPTER 24 Making the Transition from Student to Professional Nurse

View Set

Chapter 44- Management of Patients with Biliary Disorders

View Set

QP in Healthcare Chp 7: Measuring Customer Satisfaction

View Set

Assessment: Assessing Male Genitalia and Rectum

View Set

Organizational Behavior Test 1 Prof Wagner

View Set

Cultural Anthropology : Quiz 2 - Culture

View Set